Sherie bought a beach towel that normally costs $44 but was on sale for half price. She also bought a beach hat that was on sale for $12 off. Before tax, her total cost was $38. What was the regular price of the beach hat?
$22
$28
$48
$56

Answers

Answer 1

Answer:

$28

Step-by-step explanation:

The towel was $44 but at 1/2 off would be $22.

Her total cost was $38 so we subtract $22 from $38 to find how much she paid for the beach hat.

$38-$22 = $16

We add $12 to $16 to find the original price of the hat.

$16+$12 = $28

Answer 2

Answer:

$28

Step-by-step explanation:


Related Questions

20 POINTS: If tan= 3/4, find the CSC I don't know the relationship between tan and csc, please explain your answer

Answers

Answer:

[tex]\csc(x)=5/3[/tex]

Step-by-step explanation:

So we know that:

[tex]\tan(x)=3/4[/tex]

Recall that tangent represents the side opposite over the side adjacent.

This means that the opposite side is 3 while the adjacent side is 4.

Therefore, by using the Pythagorean Theorem, we can solve for the third side, which is the hypotenuse:

[tex]a^2+b^2=c^2[/tex]

Substitute a for 3 and b for 4. Thus:

[tex]3^2+4^2=c^2[/tex]

Square and add:

[tex]9+16=c^2\\25=c^2[/tex]

Square root:

[tex]c=5[/tex]

Now, recall that cosecant is the reciprocal of sine. So, find sine first.

Sine is opposite over hypotenuse. From tangent, the opposite is 3 and the hypotenuse as we now know is 5. Thus:

[tex]\sin(x)=3/5[/tex]

And cosecant is the reciprocal of that. Thus:

[tex]\csc(x)=5/3[/tex]

And that's our answer :)

Find the value of n and YZ if Y is between X and Z.

XY = 5n, YZ = 2n, XZ = 91​

Answers

The value of n and YZ are 13 and 26 respectively

If Y is between X and Z, then:

XY + YZ = XZ

Given the following

XY = 5n

YZ = 2n

XZ = 91​

Substitute the given expressions into the formula as shown:

5n + 2n = 91

7n = 91

Divide both sides by 7

7n/7 = 91/7

n = 13

Get the measure of segment YZ

YZ = 2n

YZ = 2(13)

YZ = 26

Hence the value of n and YZ are 13 and 26 respectively

Learn more here: https://brainly.com/question/23423083

Which of the following circumstances would likely make completing the square th
best method for solving a quadratic equation?
The leading coefficient is 1 and the linear term coefficient is an even number
The leading coefficient is not 1 and the constant is large
The difference of 2 perfect squares
The difference of 2 perfect cubes

Answers

Answer:

The leading coefficient is 1 and the linear term coefficient is an even number

Step-by-step explanation:

Leading coefficient must be one for the application of completing square method. If the completing square is not one than we must apply other methods like Ac method, quadratic formula to solve the equation. To solve the equation with completing square the numerical value must be on other side of the equation. In this method we multiply linear term’s coefficient with one-half and square it therefore, completing square is suitable method for the equation having leading coefficient is 1 and linear coefficient is even.

how many fewer alligators were in bite swamp than in chomp lake and reptile creek combined

Answers

Answer:

4

Step-by-step explanation:

ik it

Answer:

The answer is 4

Step-by-step explanation:

The answer is 4 because i did it on khan and it was right.

Select the correct answer

Answers

Answer:

D.  5 weeks

Step-by-step explanation:

A population of pea aphids double every week beginning with 300 aphids.

Week 1:   300 × 2 = 600

Week 2:  600 × 2 = 1200

Week 3:  1200 × 2 = 2400

Week 4:  2400 × 2 = 4800

Week 5:  4800 × 2 = 9600

It will take 5 weeks

Find the lowest number divisible by 15, 20 & 25.

Answers

Answer:

300.

Step-by-step explanation:

This is the LCM.

15  = 3 * 5

20 = 2 * 2 * 5

25 = 5 * 5

One 5 appears in all 3  factors so we only use this once in the calculation:

LCM = 2 * 2 * 3 * 5* 5 = 300

Let f(x) = 4x + 8 and g(x) = 2x - 12. Perform the function operation, (f+g)
(x), and then find the domain of the result.??

Answers

you just add these functions:

(f+g)(x)=4x+8+2x-12=6x-4

domain of this function is still all reals (R)

adam bought a box of fruit that weighed 6 1/4 pounds. if he bought a second box that weighed 8 1/8 pounds , what is the combined weight of the boxes?

Answers

Answer:

14 3/8 pounds

Step-by-step explanation:

6 pounds plus 8 pounds= 14 pounds. 1/4 + 1/8= 3/8. 1/4 can be turned into 2/8, and 2/8 plus 1/8 equals 3/8. Hope this helps :)

I’m being timed help!!!!!!!!!!!!!!

Answers

Answer: light blue

Step-by-step explanation: Ax + By = C

Answer:

3x - y = -2

Step-by-step explanation:

Assuming these rates continue, how many likes does Dagogo get, on average, for each minute of video he uploads?​

Answers

Answer:

2000

Step-by-step explanation:

Khan Academy :)

Answer:

2000

Step-by-step explanation:

bc

evaluate 3+jk+k^3 when j=2 and k=6

Answers

Answer:

[tex]231[/tex]

Step-by-step explanation:

[tex]3 + jk + k {}^{3} = 3 + 2(6) + 6 {}^{3} = 3 + 12 + 216 = 231 [/tex]

Answer:

231

Step-by-step explanation:

[tex]3+jk+k^3\\\\j = 2\\k=6\\\\3+ 2(6) + 6^3\\\\\mathrm{Follow\:the\:PEMDAS\:order\:of\:operations}\\\\\mathrm{Calculate\:exponents}\:6^3\::\quad 216\\=3+2\left(6\right)+216\\\\\mathrm{Multiply\:and\:divide\:\left(left\:to\:right\right)}\:2\left(6\right)\::\quad 12\\=3+12+216\\\\\mathrm{Add\:and\:subtract\:\left(left\:to\:right\right)}\:3+12+216\:\\:\quad 231[/tex]

Between 10pm and 7:45am, the water level in a swimming pool decreased by 3/8 in. Assuming that the water decreased at a constant rate, how much did the water level drop each hour?

Answers

Answer:

[tex]\frac{1}{26}[/tex] inches

Step-by-step explanation:

In order to solve this problem, we need to create a ratio.

Since we're looking for the rate per hour, we need to find how long it is from 10pm to 7:45am.

10pm + 2 hours = 12 AM

12 AM + 7.75 hours = 7:45 AM

So in total, it's 9.75 hours.

[tex]\frac{3}{8}[/tex] is the same as 0.375.

Our fraction will become [tex]\frac{0.375}{9.75}[/tex], which can simplify down to [tex]\frac{1}{26}[/tex].

Now we can create the proportion:

[tex]\frac{1}{26}=\frac{x}{1}[/tex]

Now we solve for x by cross multiplying.

[tex]1\cdot1=1\\\\1\div26=\frac{1}{26}[/tex]

Hope this helped!

how many fewer alligators were in bite swamp than in chomp lake and reptile creek combined

Answers

Answer:

4

Step-by-step explanation:

Solve 48=4b-4(4b-3) and show work.

Answers

Answer:

Step-by-step explanation:

4b - 16b + 12 = 48

-12b + 12 = 48

-12b = 36

b = -3

Using the digits 0-9, at most one time each, create five ordered pair that represent a linear
function that has a greater rate of change than the following (explain how u got that answer)

Answers

Answer:

(-9, 0), (-8, 1), (-7, 2), (-6, 3) and (-5, 4)

Step-by-step explanation:

From the graph attached,

Ordered pairs which lie on the given line (-9, 0), (0, 3), (3, 4), (6, 5) and (9, 6).

Since all the points lie on the same line, rate of change of the linear function will be defined by the slope of the given line.

Slope of the line passing through (0, 3) and (3, 4) = [tex]\frac{y_2-y_1}{x_2-x_1}[/tex]

                                                                                  = [tex]\frac{4-3}{3-0}[/tex]

                                                                             [tex]m_1[/tex] = [tex]\frac{1}{3}[/tex]

Now we have to find the ordered pairs which represent a linear function having slope greater than [tex]\frac{1}{3}[/tex].

So the points will be (-9, 0), (-8, 1), (-7, 2), (-6, 3) and (-5, 4).

Slope of the linear function passing through (-9, 0) and (-8, 1)

[tex]m_2[/tex] = [tex]\frac{y_2-y_1}{x_2-x_1}[/tex]

    = [tex]\frac{1-0}{-8+9}[/tex]

    = 1

Here [tex]m_2>m_1[/tex]

the data set represents the shoe sizes of 19 students in a fifth grade physical education class 4, 5, 5, 5, 6, 6, 6, 6, 7, 7, 7, 7, 7.5, 7.5, 8, 8, 8.5, 8.5, 9 a. create a box plot to represent the distribution of the data

Answers

Answer:

The minimum value = 4

The maximum value = 9

The first quartile, Q₁ = 6

The second quartile, (median) Q₂ =  7

The third quartile, Q₃ = 8

The box plot showing the above data is attached

Step-by-step explanation:

The given data set is 4, 5, 5, 5, 6, 6, 6, 6, 7, 7, 7, 7, 7.5, 7.5, 8, 8, 8.5, 8.5, 9

From the data, we have;

The minimum value = 4

The maximum value = 9

The first quartile, Q₁ = (n + 1)/4th term = (19 + 1)/4 = the 5th term = 6

The second quartile, (median) Q₂ = (n + 1)/2 term = (19 + 1)/2 = the 10th term = 7

The third quartile, Q₃ = 3×(n + 1)/4th term = 3×(19 + 1)/4 = the 15th term = 8

Please find the attached box plot as plotted on Excel.

Express in simplest radical form.
Sqrt (27)

Answers

Answer:

3√3

Step-by-step explanation:

√27 =

= √3×9

= √3×3²

= 3√3

Simplify each expression by combining like
terms.
21a + (-18b) - 6a + 1lb

Answers

Answer:

My answer is

21a-18b-6a+11b.

which is 21a-6a-18b+11b.

=15a-7b.

Find the area of the shaded region. All measurements are in centimeters. Show your reasoning. Place your answer in the box to the right.

Answers

The answer is 1,486 meters

Df = 9x -39 find EF.
DE =47

Answers

Greetings from Brasil....

Looking at the figure we can say that:

DF = DE + EF

DF = 9X - 39

DE = 47

EF = 3X + 10

inserting these data in the 1st expression....

DF = DE + EF

9X - 39 = 47 + (3X + 10)

9X - 39 = 47 + 3X + 10

9X - 3X = 57 + 39

6X = 96

X = 96/6

X = 16

EF = 3X + 10 anb X = 16, so

EF = 3·16 + 10

EF = 48 + 10

EF = 58

Mia climbs 2/5 of the height of the rock wall. Lee climbs 3/8 of Mia's distance. What fraction of the wall did Lee climb?

Answers

Answer:

you got this

Step-by-step explanation: be a smart person like you are

A cyclist rides her bike at a rate of 21 kilometers per hour. What is this rte in kilometers per minute? How many kilometers will the cyclist travel in 5 minutes? Do not round your answers.

Answers

Answer: 0.35km per minute

Step-by-step explanation:

Divide by 60 because it was in "km per HOUR".

is √13 greater or less than 3.5

Answers

Answer:yes it is greater

Step-by-step explanation:

Step-by-step explanation:

Hey, there!!

[tex] \sqrt{13 \: } \: is \: not \: greator \: than \: 3.5[/tex]

Because, root 13 means 3.60 also,

So, when you see 3.5 and 3.6 then it's obvious that 3.6 is greator.

Or if you see it in fractional form.

it will be,

18/5 and 7/2.

when we take a like term,

18×2/10 and 7×5/10

They will be ,

36/10 and 35/10.

Which implies 36/10>35/10.

So, the answer is root 13 is not greator than 3.5.

Hope it helps....

What is the degree for 6p-8

Answers

The degree is one bcz p is not having and power
Hope it helps :)

The price of a share of stock started the day at $37. During the day it went down $3, up $1, down $7 and up $4. What was the price of a share at the end of the day

Answers

Answer:

32

Step-by-step explanation:

$37 - 3 = 34
$34 + 1 = 35
$35 - 7 = 28
$28 + 4 = $32
The price at the end of the day would be $32.
:) hope you get a good grade.

A round wading pool has a diameter of 115\text{ cm}115 cm115, start text, space, c, m, end text and is 25\text{ cm}25 cm25, start text, space, c, m, end text deep. A hose is filling the pool at a rate of 34{,}000\text{ cm}^334,000 cm 3 34, comma, 000, start text, space, c, m, end text, cubed per minute. How long will it take to fill the pool to a depth of 20\text{ cm}20 cm20, start text, space, c, m, end text?

Answers

Answer: 6

Step-by-step explanation:

Answer:

6 minutes

Step-by-step explanation:

got answer from Khan and ml78786maggie

Find X. Need help ASAP

Answers

Answer:

A

Step-by-step explanation:

Given a tangent and a secant from an external point to the circle, then

the product of the external part and the whole of the secant is equal to the square of the tangent, that is

48(48 + x) = 60² = 3600 ( divide both sides by 48 )

48 + x = 75 ( subtract 48 from both sides )

x = 27 → A

dear friends answer is

x= 27

please helpp
me with this question and thank you

Answers

Answer:

[tex] \boxed{ \bold{ \huge{ \boxed{ \sf{ \frac{4x}{(x + 2)(x - 2)}}}}}} [/tex]

Step-by-step explanation:

[tex] \sf{ \frac{1}{x + 2} + \frac{1}{x - 2} + \frac{2x}{ {x}^{2} + 4} }[/tex]

Use the formula of a² - b² = ( a + b ) ( a - b )

[tex] \longrightarrow{ \sf{ \frac{1}{x + 2} + \frac{1}{x - 2} + \frac{2x}{ {x}^{2} + {2}^{2} } }}[/tex]

[tex] \longrightarrow{ \sf{ \frac{1}{x + 2} + \frac{1}{x - 2} + \frac{2x}{(x + 2)(x - 2)}}} [/tex]

[tex] \longrightarrow{ \sf{ \frac{x - \cancel{2} + x + \cancel{ 2} + 2x}{(x + 2)(x -2 )} }}[/tex]

[tex] \longrightarrow{ \sf{ \frac{4x}{(x + 2)(x - 2)}} } [/tex]

Hope I helped!

Best regards! :D

Charle has 10 guests over at his house. If he gives each guest bacon, he will have given
out a total of 30 pieces of bacon. How much bacon did each guest get?

This needs to be converted into and algebraic expression plz help its due soon

Answers

Answer:

Each guest got 3. (if he doesn't have any for himself).

Step-by-step explanation:

if each guest is x

30=10x

divide by 10

x=3

hope this helps and if you want please consider giving me brainliest! :)

A savings account rounds the amount of money deposited to the nearest hundredths place. After tax returns on April 15th, Mulan earned
exactly $239.1639 in retums.
How much money was deposited into the savings account?
A. $200
B. $239.16
C. 236.17
D. 239.164
E. I don’t know yet

Answers

Answer:

Step-by-step explanation:

Other Questions
name 5 artist with unique styles Evaluate: 20/ 52^-3 Which is a possible mechanism for why we observe the correlation that red cars are more dangerous than other colored cars?a. Younger people prefer red cars and have less driving experienceb. Red cars are more difficult to view in the evening and are therefore more prone to accidents in medium-light conditionsc. More sports cars are painted red and people who buy them are more risk-takingd. All of the above What did B. F. Skinner believe drove one's behaviors? (Site 1) You areadvised to wear mask,handsanitizer& follow socialdistancing. Why? give reason in long answer Which expression represents the phrase twice the quotient of 50 and a sum of a number y and 8 How would you describe the resulting molecule? A.) both polar and hydrophilic B.) PolarC.) NonpolarD.) HydrophobicE.) HydrophilicF.) Both Nonpolar and hydrophobic Given that a rectangle has a length of 5/2x + 10 with a width of 5/2x + 5, formulate an expression to represents the area of the rectangle. Explain why flourine forms a negative ion more easily than chlorine. A firm has a marginal cost of $20 and charges a price of $40. The Lerner index for this firm is:________.A. 0.20. B. 0.50. C. 0.33. D. 0.75. Please help, enter your answer as a number to a power! 18 cans of soda cost $6.48. What is the cost per can in dollars? Please help, I'm in a hurry 15 and 17 pls.. 15 points!!!! Which parenting style is thought to produce children who grow up to be emotionally withdrawn,distant, and lacking in curiosity? Read the Preamble. In your words, why was this Declaration of Independence created? Scott Drilling Contractors recently issued a corporate bond on which it expects to pay interest for the next twenty years. Scott would record this as a __________ on its balance sheet. If mXWZ = 90, what is x?Right angle X W Z is divided into 2 acute angles labeled 5 x plus 5 degrees and 2 x plus 8 degrees. Find the equation of a line that is perpendicular to the line -2y+20=8x and passes through the point (7,2). 3) Walt and 23 of his friends are going on a month-longcamping trip. Bags of marshmallows come with about 38marshmallows per bag. They decide to bring 12 bags ofmarshmallows to the trip. How many marshmallows willeach person get if they each eat the same number ofmarshmallows and eat all of the marshmallows at camp? rewrite expression in the form k*z^n